LSAT and Law School Admissions Forum

Get expert LSAT preparation and law school admissions advice from PowerScore Test Preparation.

 Administrator
PowerScore Staff
  • PowerScore Staff
  • Posts: 8917
  • Joined: Feb 02, 2011
|
#47088
Complete Question Explanation
(The complete setup for this game can be found here: lsat/viewtopic.php?t=11700)

The correct answer choice is (A)

This is an interesting question stem, one that asks you to substitute a new condition for the fourth rule and still achieve the same effect. Thus, you must suspend the fourth rule, and then find an answer that replaces that rule and leads to or justifies an identical result.

The best way to solve a question of this type is to examine the direct effects of the rule on the variables addressed in the suspended rule, as well as the other characteristics of the variables affected by the rule. In addition, you must guard against additional limitations posed by an answer choice. Any new limitation that results from an answer will cause that answer to be incorrect.

In this case, the fourth rule addresses T only, so this rule is easier to analyze than some of the other rules. The fourth rule established that T could not be the appointment scheduled for the sixth day. That limitation was quite localized, and lead to the inference that G, L, or S had to be scheduled for the sixth day. T was still able to move freely over the first, fourth, and fifth days (the third rule eliminated T from the second and third days, and that rule is still in effect). Thus, we need an answer that matches the limitation on the sixth day while still allowing T no limitations on the first, fourth, and fifth days.

Answer choice (A): This is the correct answer choice. This answer choice properly limits T from being scheduled for the sixth day, but imposes no other limitation on T’s movement.

Answer choice (B): This answer choice limits T to the fourth or fifth days. Thus, while it properly eliminates T from being scheduled on the sixth day, it improperly eliminates T from being scheduled on the first day. Thus, this answer choice is incorrect.

Answer choice (C): The condition in this question stem can be diagrammed as:
  • ..... ..... ..... ..... ..... ..... T :longline: L
While this eliminates T from being scheduled on the sixth day and still allows L to be scheduled on the sixth day, T is improperly limited because, for example, if L is second then T is forced into the first day. The fourth rule had no such additional limitation, and thus this answer choice is incorrect.

Answer choice (D): While this answer choice eliminates T from being scheduled for the sixth day, it clearly adds a number of additional limitations on T by tying T to G when T is not scheduled for the first day. Thus, this answer choice is incorrect.

Answer choice (E): This is an extremely tricky answer choice, and one that is well-placed by the test makers in appearing at the end. While the condition in this answer eliminates T from being sixth and allows T to move freely among the first, fourth, and fifth days, this answer is incorrect because it establishes an extra limitation on the sixth day, namely that L cannot be scheduled for the sixth day. If only G and S can be scheduled for the sixth day, then L is also eliminated, which differs from the final setup of the game. Thus, this answer choice does not have the same effect as the fourth rule of the game, and is incorrect.
 LustingFor!L
  • Posts: 80
  • Joined: Aug 27, 2016
|
#33735
What is the best approach for these kind of questions. Usually I go rule by rule and lucked out that A was the correct answer, so I didn't waste a lot of time.
 Steven Palmer
PowerScore Staff
  • PowerScore Staff
  • Posts: 35
  • Joined: Feb 21, 2017
|
#33758
Hello,

These are probably the hardest types of Games questions. There really isn't a fast way to go about them, which is why I often recommend students not to spend much time on them unless they have finished the rest of the games and questions in the section. A good way to think about it, as you go through the options, is whether anything new could happen with the new rule that wouldn't be allowed with the rule being replaced. Conversely, you can also ask yourself whether the new rule would prevent something that would be allowed under the old rule. If these statements are true, you can cross out the answer. As you can see, pretty time consuming!

Also, I'm not sure if you have these resources, but there is a good discussion of these types of questions in the Logic Games Bible, and in the L11 Supplemental area for the full length and full live online classes.

Hope this helps!
Steven
 blade21cn
  • Posts: 100
  • Joined: May 21, 2019
|
#73006
How do we diagram "(A) The telephone appointment must be scheduled for an EARLIER day THAN the gas appointment OR the satellite appointment, or both"?

I'd go with "T < G or S," or it could be broken down into three scenarios: "T < G and S," "G-T-S," and "S-T-G."

However, on Page 2-61 of the Course Textbook, 《The Exception to the Rule: "Than Either"》: In most cases, "either/or" means "at least one of the two, possibly both"; However, when the phrase "than either" is used, the term "either" translates to "both."

Thus, according to the above, (A) should be diagrammed only as "T < G and S," i.e., "T is earlier than BOTH G AND S." But clearly, this interpretation does not work in this question, especially because (A) is the correct answer and we can have "G/S-T-S/G." So it sounds to me that the Textbook is wrong. Can anyone clarify this? Thanks!
 James Finch
PowerScore Staff
  • PowerScore Staff
  • Posts: 943
  • Joined: Sep 06, 2017
|
#73018
Hi Blade,

It seems like you're misreading (A); the terminology used is "T must be scheduled for an earlier day than the gas appointment or the satellite appointment, or both," which clearly indicates that T must come before at least one of the two variables, possibly both. As you note, this creates three scenarios: G-T-S, S-T-G, and T-S/G.

Now if the language noted in the book was used, (A) would read something like: "T must be scheduled for an earlier day than either the gas appointment or the satellite appointment," which would indicate that it would have to come before both of them, or T-S/G.

Hope this helps!
 blade21cn
  • Posts: 100
  • Joined: May 21, 2019
|
#73023
Just a quick followup, James, logically speaking, "either A or B" is different from "A or B"? I thought "either" is optional. The focus here is the usage of "or," after "than." That is, does "than ... or" equal "than ... both"?
User avatar
 Dave Killoran
PowerScore Staff
  • PowerScore Staff
  • Posts: 5852
  • Joined: Mar 25, 2011
|
#73025
blade21cn wrote:Just a quick followup, James, logically speaking, "either A or B" is different from "A or B"? I thought "either" is optional. The focus here is the usage of "or," after "than." That is, does "than ... or" equal "than ... both"?
I know James has this one covered, but I thought I'd jump in with a useful link:

https://blog.powerscore.com/lsat/bid-26 ... -the-lsat/

It's the "than either" that makes it both.

Thanks!
 angelsfan0055
  • Posts: 39
  • Joined: Feb 26, 2021
|
#85186
I picked A because I went through my previous diagrams and saw that in each case T was either before both G and S or before one of them. I wanted to examine answer choice E a little bit more. I ruled it out because as it is mentioned in the original explanation, it's possible for L to come as the final selection. This is something I noted in my initial inferences, seeing as S, G and L are the only ones that can go last. Also, from answer choice 5, L was the one I put last. Just want to make sure my thinking was correct on this one. I think I'm slowly getting these substitution q's.
User avatar
 Olivia232425
  • Posts: 1
  • Joined: Mar 10, 2021
|
#85187
I'm not sure if it can be left here. But suddenly ... the need to pay scientists more and more every year explains why, although funding has increased over the past ten years, the current amount is still not enough. Why doesn't B help resolve the paradox?
___________________________________________________
Olivia, Employee Monitoring Manager
 Adam Tyson
PowerScore Staff
  • PowerScore Staff
  • Posts: 5153
  • Joined: Apr 14, 2011
|
#85213
Hi Olivia! The thread you posted this under is about a question from the December 2010 Logic Games section, but it looks like you are asking about a Resolve the Paradox question from a Logical Reasoning section. I searched the December 2010 LR sections and didn't find a Resolve question about scientists' pay. If you could tell us which test and section this question is coming from, we can find the correct thread in this Forum and then, if the explanation provided doesn't answer your question, you could pose it again in that thread and we will follow up with you there.

Get the most out of your LSAT Prep Plus subscription.

Analyze and track your performance with our Testing and Analytics Package.